PT 31 Section 2 Question 3 Forum

Prepare for the LSAT or discuss it with others in this forum.
Post Reply
delusional

Silver
Posts: 1201
Joined: Thu Jul 15, 2010 7:57 pm

PT 31 Section 2 Question 3

Post by delusional » Wed Aug 25, 2010 8:56 pm

I'm very frustrated because I'd been getting most LR sections perfect and now suddenly, no matter how I focus on the tough questions, I get at least one wrong. Yeah, I try to learn from them and I usually agree with the right answer once I've thought about it, but then the next section I do inevitably has one that I'll get wrong, too, and I can't seem to get a section perfect anymore.

Here's one question that I got wrong in PT and I still don't agree; I think that the q and a are faulty. If you're familiar with the question, please chime in, and if you take the time to check it up, God bless! Mods - I'm trying to post this legally, and I apologize if it crosses the line.

PT 31 Section 2 Question 3

The question concerns the TV program to answer whether female physicians are more sensitive to women's needs.

Kaplan explains the answer as follows: (I bolded part of it to make my point)

The question stem identifies this as a Flaw question—we need to find out where the argument goes wrong. The TV program
is looking to determine who female patients think are more sensitive to their needs: female physicians or male
physicians. To answer this, they are going to poll the doctors. Did you recognize the fairly blatant scope shift contained in
this reasoning? If you want to know the opinion of the female patients, you have to ask the female patients, not the doctors!
Answer choice (C) reflects this precisely.


The two possible answers are:

a - it's flawed because physicians don't like to discuss other physicians' style. I don't love this answer but I chose it. The question doesn't discuss "style" per se, it discusses sensitivity. Because I didn't like the other answer, I assumed that a sensitivity could be part of the physician's style. (I guess that was the mistake.)

The other option was

c - patients are best able to determine the answer to this question rather than doctors. I disliked this answer more than (a) but it turned out to be the right one. I disliked it because, so what if patients are BEST able to answer the question? The TV program can STILL be a great idea, because you can still potentially get the answer from doctors!

Looking back at Kaplan's explanation didn't help. Notice the part that I bolded. If the question the TV program attempted to address was how PATIENTS feel about doctors' sensitivity, the answer would obviously be c. But looking back at the question, it doesn't mention patients' opinions at all. It discusses whether female doctors actually ARE more sensitive.

I know how hard it is to comment about something like this; ya gotta whip out the old book and find the question and see what you'd answer, but I'm trying to learn something, anything from this question. Or else find out that a flawed question can make it to a PT. God knows that tons of the fake LSAT PTs have awful questions, why not one on a real LSAT? Please shed some light if you can!

User avatar
EarlCat

Silver
Posts: 606
Joined: Mon Mar 12, 2007 4:04 pm

Re: PT 31 Section 2 Question 3

Post by EarlCat » Wed Aug 25, 2010 9:21 pm

First off, Kaplan's explanation is just plain wrong. As you pointed out, the stim has nothing to do with what female patients "think," so the "flaw" isn't as obvious as Kaplan would like to make it out to be. Second, and more important, this is not a flaw question.

Flaw questions ask you to find a description of the flaw that occurred in the argument. For instance, "The program's plan is flawed because it:" and the answer would be a phrase describing at least one of the mistakes. E.g. "Presumes without providing justification that physicians of both sexes are capable of determining the sensitivity of other physicians." Notice how that describes something the argument does--it makes an unsupported assumption.

This, however, is a weaken question. It is looking for a fact that, if true, makes the conclusion less likely (which it always does by indirectly calling attention to a flaw).

Now, the important thing here is that weakening facts don't have to destroy the argument. They only have to weaken it a teeny tiny little bit.

You wrote:
I disliked [C] because, so what if patients are BEST able to answer the question? The TV program can STILL be a great idea, because you can still potentially get the answer from doctors!
Yes, it may still be a great idea, but once we know that patients are better able to answer the question, we know the idea is imperfect--it's been weakened. And thus we have our answer.

edit:
If there's any doubt that it's been weakened, look at your own language. "The TV program can still be a great idea, because you can still potentially get the answer from doctors." Compare this to the stim which claimed they will "get the answer," not just "potentially get the answer." You added that hedging yourself after considering the (weakening) fact given by C.

Post Reply

Return to “LSAT Prep and Discussion Forum”